Sei sulla pagina 1di 60

1. A 19 year old female is admitted with suspected meningitis.

The House Officer is due


to perform a lumbar puncture. What is the most likely structure first encountered when
the needle is inserted?
A. Ligamentum flavum
B. Denticulate ligament
C. Dural sheath
D. Pia Mater
E. Supraspinous ligament
Theme from September 2012 exam

2. A 34 year old male is being examined in the pre-operative assessment clinic. A


murmur is identified in the 5th intercostal space just next to the left side of the sternum.
From where is it most likely to have originated?
A. Mitral valve
B. Aortic valve
C. Pulmonary valve
D. Right ventricular aneurysm
E. Tricuspid valve
Theme from September 2012 Exam

3. What is the correct embryological origin of the stapes?


A. First pharyngeal arch
B. Second pharyngeal arch
C. Third pharyngeal arch
D. Fourth pharyngeal arch
E. Fifth pharyngeal arch
Embryological origin stapes = 2nd
pharyngeal arch
Theme from April 2012 Exam

4. Which muscle initiates abduction of the shoulder?


A. Infraspinatus
B. Latissimus dorsi
C. Supraspinatus
D. Deltoid
E. Teres major
Theme from September 2012 exam

5. What is the most important structure involved in supporting the uterus?


A. Round ligament
B. Broad ligament
C. Uterosacral ligaments
D. Cardinal ligaments
E. Central perineal tendon
Theme from September 2012 Exam

1
6. A 78 year old man is due to undergo an endarterectomy of the internal carotid artery.
Which of the following nervous structures are most at risk during the dissection?
A. Recurrent laryngeal nerve
B. Sympathetic chain
C. Hypoglossal nerve
D. Phrenic nerve
E. Lingual nerve
Nerves at risk during a carotid endarterectomy:
1. Hypoglossal nerve
2. Greater auricular nerve
3. Superior laryngeal nerve
Theme from May 2011, Jan 2013 exam

7. Which of the following structures is not transmitted by the jugular foramen?


A. Hypoglossal nerve
B. Accessory nerve
C. Internal jugular vein
D. Inferior petrosal sinus
E. Vagus nerve
Contents of the jugular foramen:
1. Anterior: inferior petrosal sinus
2. Intermediate: glossopharyngeal, vagus, and accessory nerves
3. Posterior: sigmoid sinus (becoming the internal jugular vein) and some meningeal
branches from the occipital
4. and ascending pharyngeal arteries
Theme from 2009 exam

8. With regards to the jugular vein, which of the following statements is untrue?
A. It lies within the carotid sheath
B. It is the continuation of the sigmoid sinus
C. The terminal part of the thoracic duct crosses anterior to it to insert
into the right subclavian vein
D. The hypoglossal nerve is closely related to it as it passes near the atlas
E. The vagus nerve is closely related to it within the carotid sheath
Theme from April 2013 Exam

9. A 28 year old man requires a urethral catheter to be inserted prior to undergoing a


splenectomy. Where is the first site of resistance to be encountered on inserting the
catheter?
A. Bulbar urethra
B. Membranous urethra
C. Internal sphincter
D. Prostatic urethra

2
E. Bladder neck
Theme from 2011 exam
Theme from January 2012 Exam

10. A 23 year old man undergoes an orchidectomy. The right testicular vein is ligated;
into which structure does it drain?
A. Right renal vein
B. Inferior vena cava
C. Common iliac vein
D. Internal iliac vein
E. External iliac vein
Theme from April 2012 Exam

11. A 24 year old man falls and sustains a fracture through his scaphoid bone. From
which of the following areas does the scaphoid derive the majority of its blood supply?
A. From its proximal medial border
B. From its proximal lateral border
C. From its proximal posterior surface
D. From the proximal end
E. From the distal end
Theme from April 2012 Exam

12. Which of the following forms the medial wall of the femoral canal?
A. Pectineal ligament
B. Adductor longus
C. Sartorius
D. Lacunar ligament
E. Inguinal ligament
Theme from September 2011 Exam
Theme from September 2012 Exam

13. A 67 year old man is undergoing a transurethral resection of a bladder tumour using
diathermy. Suddenly during the procedure the patients leg begins to twitch. Stimulation
of which of the following nerves is the most likely cause?
A. Femoral
B. Pudendal
C. Sciatic
D. Obturator
E. Gluteal
Theme from January 2011 Exam
Theme from January 2013 Exam

3
14. A 5 year old boy is playing with some small ball bearings. Unfortunately, he inhales
one. To which of the following lung regions is the ball most likely to settle?
A. Right lower lobe
B. Left main bronchus
C. Right upper lobe
D. Left lower lobe
E. None of the above
Theme from September 2011 Exam
Theme from January 2013 Exam

15. An 18 year old man is cutting some plants when a small piece of vegetable matter
enters his eye. His eye becomes watery. Which of the following is responsible for
relaying parasympathetic neuronal signals to the lacrimal apparatus?
A. Pterygopalatine ganglion
B. Otic ganglion
C. Submandibular ganglion
D. Ciliary ganglion
E. None of the above
Theme from January 2013 Exam

16. A 43 year old lady is undergoing a total thyroidectomy for an extremely large goitre.
The surgeons decide that access may be improved by division of the infra hyoid strap
muscles. At which of the following sites should they be divided?
A. In their upper half
B. In their lower half
C. In the middle
D. At their origin from the hyoid
E. At the point of their insertion
Theme from 2009 Exam

17. A 22 year old women has recently undergone a surgical excision of the
submandibular gland. She presents to the follow up clinic with a complaint of tongue
weakness on the ipsilateral side to her surgery. Which nerve has been damaged?
A. Hypoglossal nerve
B. Lingual nerve
C. Inferior alveolar nerve
D. Facial nerve
E. Lesser petrosal nerve
Three cranial nerves may be injured during submandibular gland excision.
1. Marginal mandibular branch of the facial nerve
2. Lingual nerve
3. Hypoglossal nerve
Theme from April 2012 Exam

4
18. You decide to take an arterial blood gas from the femoral artery. Where should the
needle be inserted to gain the sample?
A. Mid point of the inguinal ligament
B. Mid inguinal point
C. 2cm inferomedially to the pubic tubercle
D. 2cm superomedially to the pubic tubercle
E. 3cm inferolaterally to the deep inguinal ring
Theme from September 2012 Exam

19. A 67 year old man undergoes a carotid endarterectomy and seems to recover well
following surgery. When he is reviewed on the ward post operatively he complains that
his voice is hoarse. What is the most likely cause?
A. Damage to the accessory nerve
B. Damage to the cervical plexus
C. Damage to the glossopharyngeal nerve
D. Damage to the hypoglossal nerve
E. Damage to the vagus
Theme from April 2013 Exam

20. A 25 year old man has an inguinal hernia, which of the following structures must be
divided (at open surgery) to gain access to the inguinal canal?
A. Transversalis fascia
B. External oblique aponeurosis
C. Conjoint tendon
D. Rectus abdominis
E. Inferior epigastric artery
Theme from January 2013 Exam

21. A 34 year old man is shot in the postero- inferior aspect of his thigh. Which of the
following lies at the most lateral aspect of the popliteal fossa?
A. Popliteal artery
B. Popliteal vein
C. Common peroneal nerve
D. Tibial nerve
E. Small saphenous vein
Theme from April 2012 Exam

22. A 67 year old man has an abdominal aortic aneurysm which displaces the left renal
vein. Which branch of the aorta is most likely to affected at this level?
A. Inferior mesenteric artery
B. Superior mesenteric artery
C. Coeliac axis
D. Testicular artery
E. None of the above
Theme from April 2013 Exam

5
23. A 12 year old boy undergoes surgery for recurrent mastoid infections. Post
operatively he complains of an altered taste sensation. Which of the following nerves has
been injured?
A. Glossopharyngeal
B. Greater petrosal
C. Olfactory
D. Trigeminal
E. Chorda tympani
Theme from April 2012 Exam

24. What is the anatomical level of the transpyloric plane?


A. T11
B. T12
C. L1
D. L4
E. T10
Theme from September 2012 Exam
Theme from January 2013 Exam

25. A 35 year old man falls and sustains a fracture to the middle third of his clavicle.
Which vessel is at greatest risk of injury?
A. Subclavian vein
B. Subclavian artery
C. External carotid artery
D. Internal carotid artery
E. Vertebral artery
Theme from April 2013 Exam

26. A 33 year old man sustains an injury to his forearm and wrist. When examined in
clinic he is unable to adduct his thumb. What is the most likely underlying nerve lesion?
A. Radial nerve
B. Superficial branch of the ulnar nerve
C. Median nerve
D. Posterior interosseous nerve
E. Deep branch of the ulnar nerve
Theme from April 2013 Exam

27. A 63 year old man who smokes heavily presents with dyspepsia. He is tested and
found to be positive for helicobacter pylori infection. One evening he has an episode of
haematemesis and collapses. What is the most likely vessel to be responsible?
A. Portal vein
B. Short gastric arteries
C. Superior mesenteric artery
D. Gastroduodenal artery
E. None of the above
Theme from January 2012 exam

6
28. A 22 year old man is stabbed in the chest at the level of the junction between the
sternum and manubrium. Which structure is at greatest risk?
A. Left atrium
B. Oesophagus
C. Thyroid gland
D. Inferior vena cava
E. Aortic arch
Theme from 2009 Exam

29. Which of the following nerves passes through the greater and lesser sciatic foramina?
A. Pudendal nerve
B. Sciatic nerve
C. Superior gluteal nerve
D. Inferior gluteal nerve
E. Posterior cutaneous nerve of the thigh
Structures passing through the lesser and greater sciatic foramina (medial to lateral): PIN
1. Pudendal nerve
2. Internal pudendal artery
3. Nerve to obturator internus
Theme from 2010 Exam, Theme from January 2013 Exam

30. A 73 year old lady is admitted with brisk rectal bleeding. Despite attempts at
resuscitation the bleeding proceeds to cause haemodynamic compromise. An upper GI
endoscopy is normal. A mesenteric angiogram is performed and a contrast blush is seen
in the region of the sigmoid colon. The radiologist decides to embolise the vessel
supplying this area. At what spinal level does it leave the aorta?
A. L2
B. L1
C. L4
D. L3
E. T10
Theme from 2009 Exam

31. In which space is a lumbar puncture performed?


A. Subdural space
B. Epidural space
C. Subarachnoid space
D. Extradural space
E. Intraventricular space
Theme from January 2013 exam

32. A 21 year old man is stabbed in the antecubital fossa. A decision is made to surgically
explore the wound. At operation the surgeon dissects down onto the brachial artery. A
nerve is identified medially; which nerve is it likely to be?
A. Radial

7
B. Recurrent branch of median
C. Anterior interosseous
D. Ulnar
E. Median
Theme from September 2012 Exam

33. A 65 year old man with long standing atrial fibrillation develops an embolus to the
lower leg. The decision is made to perform an embolectomy, utilising a trans popliteal
approach. After incising the deep fascia, which of the following structures will the
surgeons encounter first on exploring the central region of the popliteal fossa?
A. Popliteal vein
B. Common peroneal nerve
C. Popliteal artery
D. Tibial nerve
E. None of the above
Theme from 2009 Exam, Theme from January 2013 Exam

34. A man sustains a laceration between the base of the little finger and wrist. Several
weeks after the injury there is loss of thumb adduction power. Which nerve is most likely
to have been injured?
A. Superficial ulnar nerve
B. Deep ulnar nerve
C. Median nerve
D. Radial nerve
E. Recurrent branch of median nerve
Theme from 2009 Exam

35. What is the course of the median nerve relative to the brachial artery in the upper
arm?
A. Medial to anterior to lateral
B. Lateral to posterior to medial
C. Medial to posterior to lateral
D. Medial to anterior to medial
E. Lateral to anterior to medial
Relations of median nerve to the brachial artery: Lateral -> Anterior -> Medial
Theme from 2009 and 2012 Exams

36. A 43 year old man is due to undergo an excision of the sub mandibular gland. Which
of the following incisions is the most appropriate for this procedure?
A. A transversely orientated incision 4cm below the mandible
B. A transversely orientated incision immediately inferior to the mandible
C. A vertical incision 3 cm anterior to the angle of the mandible and extending
inferiorly
D. A transversely orientated incision 2cm above the mandible
E. A transversely orientated incision 12cm below the mandible
Theme from 2009 Exam

8
37. A 5 year old boy presents with recurrent headaches. As part of his assessment he
undergoes an MRI scan of his brain. This demonstrates enlargement of the lateral and
third ventricles. Where is the most likely site of obstruction?
A. Foramen of Luschka
B. Foramen of Magendie
C. Foramen of Munro
D. Aqueduct of Sylvius
E. None of the above
Theme based on September 2011, April 2012, September 2012 Exam

38. Where does the spinal cord terminate in neonates?


A. L1
B. L2
C. L3
D. L4
E. L5
Theme from 2009, January 2013 Exam

39. A 73 year old lady is admitted with acute mesenteric ischaemia. A CT angiogram is
performed and a stenotic lesion is noted at the origin of the superior mesenteric artery. At
which of the following levels does this branch from the aorta?
A. L1
B. L2
C. L3
D. L4
E. L5
Theme from January 2012 Exam

40. Following an oesophagogastrectomy the surgeons will anastomose the oesophageal


remnant to the stomach, which of the following is not part of the layers that comprise the
oesophageal wall?
A. Serosa
B. Adventitia
C. Muscularis propria
D. Submucosa
E. Mucosa
Theme from April 2010 exam

41. Where is the 'safe triangle' for chest drain insertion located?
A. 4th intercostal space, mid axillary line
B. 5th intercostal space, mid axillary line
C. 4th intercostal space, mid scapular line
D. 5th intercostal space, mid scapular line
E. 4th intercostal space, mid clavicular line
'Safe Triangle' for chest drain insertion: 5th intercostal space, mid axillary line, April
2012 exam

9
42. Your consultant decides to perform an open inguinal hernia repair under local
anaesthesia. Which of the following dermatomal levels will require blockade?
A. T10
B. T12
C. T11
D. S1
E. S2
Theme from April 2012 Exam

43. Which of the following does not pass through the superior orbital fissure?
A. Inferior opthalmic vein
B. Abducens nerve
C. Opthalmic artery
D. Trochlear nerve
E. Superior opthalmic vein
Mnemonic for the nerves passing through the supraorbital fissure: Live Frankly To See
1. Absolutely No Insult
2. Lacrimal
3. Frontal
4. Trochlear
5. Superior Division of
6. Oculomotor
7. Abducens
8. Nasociliary
9. Inferior Division of
10. Oculomotor nerve
Theme from January 2012 Exam

44. A patient is found to have an ischaemic left colon. Which artery arising from the aorta
at around the level of L3 is most likely to account for this situation?
A. Superior mesenteric artery
B. Inferior mesenteric artery
C. Superior rectal artery
D. Ileocolic artery
E. Middle colic artery
Theme from January 2013 Exam

45. At which level does the aorta perforate the diaphragm?


A. T10
B. T9
C. T8
D. T11
E. T12
Memory aid:
T8 (8 letters) = vena cava, T10 (10 letters) = esophagus, T12 (12 letters) = aortic
hiatus (Theme from April 2012 exam)

10
46. A 24 year old lady is stabbed in the buttock. Following the injury the wound is
sutured in the emergency department. Eight weeks later she attends the clinic, as she
walks into the clinic room she has a waddling gait and difficulty with thigh abduction. On
examination she has buttock muscle wasting. Which nerve has been injured?
A. Superior gluteal nerve
B. Obturator nerve
C. Sciatic nerve
D. Femoral nerve
E. Inferior gluteal nerve
Theme from April 2012 Exam

47. At which level is the hilum of the left kidney located?


A. L1
B. L2
C. T12
D. T11
E. L3
Remember L1 ('left one') is the level of the hilum of the left kidney
Theme from April 2012 exam

48. A 73 year old lady presents with symptoms of faecal incontinence. On examination
she has weak anal sphincter muscles. What are the main nerve root values of the nerves
supplying the external anal sphincter?
A. S2,3
B. L5, S1
C. S4,5
D. S5
E. S2,3,4
Theme from September 2011 Exam

49. A 72 year old man has a fall. He is found to have a fractured neck of femur and goes
on to have a left hip hemiarthroplasty. Two months post operatively he is found to have
an odd gait. When standing on his left leg his pelvis dips on the right side. There is no
foot drop. What is the cause?
A. Sciatic nerve damage
B. L5 radiculopathy
C. Inferior gluteal nerve damage
D. Previous poliomyelitis
E. Superior gluteal nerve damage
Theme from 2010 Exam

50. A 28 year old lady requires an episiotomy for a ventouse vaginal delivery. Which of
the nerves listed below will usually be anaesthetised to allow the episiotomy?
A. Femoral
B. Ilioinguinal
C. Pudendal

11
D. Genitofemoral
E. Sacral plexus
Theme from April 2013 Exam

51. A 48 year old lady is undergoing an axillary node clearance for breast cancer. Which
of the structures listed below are most likely to be encountered during the axillary
dissection?
A. Cords of the brachial plexus
B. Thoracodorsal trunk
C. Internal mammary artery
D. Thoracoacromial artery
E. None of the above
Beware of damaging the thoracodorsal trunk if a latissimus dorsi flap reconstruction is
planned. Theme from 2009 Exam

53. A 53 year old lady is recovering following a difficult mastectomy and axillary nodal
clearance for carcinoma of
the breast. She complains of shoulder pain and on examination has obvious winging of
the scapula. Loss of
innervation to which of the following is the most likely underlying cause?
A. Latissimus dorsi
B. Serratus anterior
C. Pectoralis minor
D. Pectoralis major
E. Rhomboids
Theme from April 2012 Exam

54. A 56 year old lady is referred to the colorectal clinic with symptoms of pruritus ani.
On examination a polypoidal mass is identified inferior to the dentate line. A biopsy
confirms squamous cell carcinoma. To which of the following lymph node groups will
the lesion potentially metastasise?
A. Internal iliac
B. External iliac
C. Mesorectal
D. Inguinal
E. None of the above
Theme from September 2011, April 2012 Exam

55. A 20 year old man is hit with a hammer on the right side of the head. He dies on
arrival in the emergency department. Which of these features is most likely to be found at
post mortem? Theme based on 2011 exam
A. Hydrocephalus
B. Supra tentorial herniation
C. Laceration of the middle meningeal artery
D. Sub dural haematoma
E. Posterior fossa haematoma

12
56. Which of the following ligaments contains the artery supplying the head of femur in
children?
A. Transverse ligament
B. Ligamentum teres
C. Iliofemoral ligament
D. Ischiofemoral ligament
E. Pubofemoral ligament
Theme from 2010 Exam

57. A 63 year old man is undergoing a coronary artery bypass procedure. During the
median sternotomy which structure would routinely require division?
A. Parietal pleura
B. Interclavicular ligament
C. Internal mammary artery
D. Brachiocephalic vein
E. Left vagus nerve
Theme from January 2011 Exam

58. A man undergoes a high anterior resection for carcinoma of the upper rectum. Which
of the following vessels will require ligation?
A. Superior mesenteric artery
B. Inferior mesenteric artery
C. Coeliac axis
D. Perineal artery
E. Middle colic artery
Theme from April 2013

59. What are the boundaries of the 'safe triangle' for chest drain insertion?
A. Bounded by trapezius, latissimus dorsi, and laterally by the vertebral border of
the scapula
B. Bounded by latissimus dorsi, pectoralis major, line superior to the nipple and
apex at the axilla
C. Bounded by latissimus dorsi, serratus anterior, line superior to the nipple and
apex at the axilla
D. Bounded by trapezius, deltoid, rhomboid major and teres minor
E. Bounded by trapezius, deltoid and latissimus dorsi
Theme from April 2012 Exam

60. A 60 year old female attends the preoperative hernia clinic. She reports some visual
difficulty. On examination she is noted to have a homonymous hemianopia. Where is the
lesion most likely to be?
A. Frontal lobe
B. Pituitary gland
C. Parietal lobe
D. Optic chiasm
E. Optic tract

13
Lesions before optic chiasm:
Monocular vision loss = Optic nerve lesion
Bitemporal hemianopia = Optic chiasm lesion
Lesions after the optic chiasm:
Homonymous hemianopia = Optic tract lesion
Upper quadranopia = Temporal lobe lesion
Lower quadranopia = Parietal lobelesion
Theme from April 2012 exam

61. A 23 year old climber falls and fractures his humerus. The surgeons decide upon a
posterior approach to the middle third of the bone. Which of the following nerves is at
greatest risk in this approach?
A. Ulnar
B. Antebrachial
C. Musculocutaneous
D. Radial
E. Intercostobrachial
Theme from April 2012 Exam

62. Following a carotid endarterectomy a man notices that he has a weakness of his
tongue. Damage to which of the following nerves is the most likely explanation for this
process?
A. Hypoglossal
B. Accessory
C. Ansa cervicalis
D. Vagus
E. Cervical plexus
Theme from April 2013

63. At which of the following levels does the inferior vena cava exit the abdominal
cavity?
A. T6
B. T7
C. T10
D. T8
E. T12
Theme from April 2012 Exam

64. Which of the following structures lies deepest in the popliteal fossa?
A. Popliteal artery
B. Popliteal vein
C. Tibial nerve
D. Common peroneal nerve
E. Popliteal lymph nodes
Theme from January 2012 exam

14
65. What is the nerve root value of the external urethral sphincter?
A. S4
B. S1, S2, S3
C. S2, S3, S4
D. L3, L4, L5
E. L5, S1, S2
Theme from April 2012 Exam

66. A patient presents to the clinic following a surgical procedure. She complains that she
is unable to shrug her shoulder. What is the most likely underlying nerve injury?
A. Accessory nerve
B. Cervical plexus
C. Ansa cervicalis
D. Long thoracic nerve
E. Axillary nerve
Theme from April 2013 Exam

67. A 32 year old man is undergoing a splenectomy. Division of which of the following
will be necessary during the procedure?
A. Left crus of diaphragm
B. Short gastric vessels
C. Gerotas fascia
D. Splenic flexure of colon
E. Marginal artery
Theme from 2011 Exam

68. A patient has a chest drain insertion. There is fresh blood at the chest drain insertion
area. Which vessel has been damaged?
A. Pericardiophrenic artery
B. Intercostal vein
C. Right ventricle
D. Vagus artery
E. Intercostal artery
Theme from 2009 Exam

69. Two teenagers are playing with an airgun when one accidentally shoots his friend in
the abdomen. He is brought to the emergency department. On examination there is a
bullet entry point immediately to the right of the rectus sheath at the level of the 1st
lumbar vertebra. Which of the following structures is most likely to be injured by the
bullet?
A. Head of pancreas
B. Right ureter
C. Right adrenal gland
D. Fundus of the gallbladder
E. Gastric antrum
Theme from September 2011 Exam

15
70. Which of the following nerves is not contained within the posterior triangle of the
neck?
A. Accessory nerve
B. Phrenic nerve
C. Greater auricular nerve
D. Ansa cervicalis
E. Lesser occiptal nerve
Theme from September 2012 Exam

71. A 42 year old lady is reviewed in the outpatient clinic following a routine surgical
procedure. She complains of diminished sensation at the lateral aspect of her foot. Which
of the following nerves is likely to be affected?
A. Sural
B. Superficial peroneal
C. Deep peroneal
D. Medial plantar
E. Lateral plantar
Theme from April 2012 Exam

72. A sprinter attends A&E with severe leg pain. He had forgotten to warm up and ran a
100m sprint race. Towards the end of the race he experienced pain in the posterior aspect
of his thigh. The pain worsens, localising to the lateral aspect of the knee. The sprinter is
unable to flex the knee. What structure has been injured?
A. Anterior cruciate ligament
B. Posterior cruciate ligament
C. Semimembranosus tendon
D. Semiteninosus tendon
E. Biceps femoris tendon
Theme from 2009 Exam

73. Which nerve supplies the interossei of the fourth finger?


A. Radial
B. Median
C. Superficial ulnar
D. Deep ulnar
E. Posterior interosseous

Mnemonic:
PAD and DAB
Palmer interossei ADduct
Dorsal interossei ABuct
Theme from April 2013 Exam

16
74. A 45 year man presents with hand weakness. He is given a piece of paper to hold
between his thumb and index finger. When the paper is pulled, the patient has difficulty
maintaining a grip. Grip pressure is maintained by flexing the thumb at the
interphalangeal joint. What is the most likely nerve lesion?
A. Posterior interosseous nerve
B. Deep branch of ulnar nerve
C. Anterior interosseous nerve
D. Superficial branch of the ulnar nerve
E. Radial nerve
Theme from January 2012 exam

75. Which of the following cranial foramina pairings are incorrect?


A. The foramen lacerum and internal carotid artery.
B. Foramen ovale and mandibular nerve.
C. Optic canal and ophthalmic artery.
D. Optic canal and ophthalmic nerve.
E. Foramen rotundum and maxillary nerve.
Question derived from 2010 and 2011 exams

76. A man has an incision sited than runs 8cm from the deltopectoral groove to the
midline. Which of the following is not at risk of injury?
A. Cephalic vein
B. Shoulder joint capsule
C. Axillary artery
D. Pectoralis major
E. Trunk of the brachial plexus
Theme from April 2012 Exam

77. A surgeon is due to perform a laparotomy for perforated duodenal ulcer. An upper
midline incision is to be performed. Which of the following structures is the incision
most likely to divide?
A. Rectus abdominis muscle
B. External oblique muscle
C. Linea alba
D. Internal oblique muscle
E. None of the above
Theme from September 2011 Exam

78. A 23 year old man is injured during a game of rugby. He suffers a fracture of the
distal third of his clavicle, it is a compound fracture and there is evidence of arterial
haemorrhage. Which of the following vessels is most likely to be encountered first during
subsequent surgical exploration?
A. Posterior circumflex humeral artery
B. Axillary artery
C. Thoracoacromial artery
D. Sub scapular artery

17
E. Lateral thoracic artery
Similar theme in September 2011 Exam

79. What is the most useful test to clinically distinguish between an upper and lower
motor neurone lesion of the facial nerve?
A. Blow cheeks out
B. Loss of chin reflex
C. Close eye
D. Raise eyebrow
E. Open mouth against resistance
Upper motor neurone lesions of the facial nerve- Paralysis of the lower half of face.
Lower motor neurone lesion- Paralysis of the entire ipsilateral face.
Theme from April 2012 Exam

80. A 23 year old man is involved in a fight, during the dispute he sustains a laceration to
the posterior aspect of his right arm, approximately 2cm proximal to the olecranon
process. On assessment in the emergency department he is unable to extend his elbow
joint. Which of the following tendons is most likely to have been cut?
A. Triceps
B. Pronator teres
C. Brachioradialis
D. Brachialis
E. Biceps
Theme from 2009 Exam

81. During the course of a radical gastrectomy the surgeons detach the omentum and
ligate the right gastro-epiploic artery. From which vessel does it originate?
A. Superior mesenteric artery
B. Inferior mesenteric artery
C. Coeliac axis
D. Common hepatic artery
E. Gastroduodenal artery
Theme from January 2013 Exam

82. A 43 year old man is reviewed in the clinic following a cardiac operation. A chest x-
ray is performed and a circular radio-opaque structure is noted medial to the 4th
interspace on the left. Which of the following procedures is the patient most likely to
have undergone?
A. Aortic valve replacement with metallic valve
B. Tricuspid valve replacement with metallic valve
C. Tricuspid valve replacement with porcine valve
D. Pulmonary valve replacement with porcine valve
E. Mitral valve replacement with metallic valve
Theme from April 2012 Exam

18
83. A 63 year old lady is diagnosed as having an endometrial carcinoma arising from the
uterine body. To which nodal region will the tumour initially metastasise?
A. Para aortic nodes
B. Iliac lymph nodes
C. Inguinal nodes
D. Pre sacral nodes
E. Mesorectal lymph nodes
Theme from 2011 exam

84. A 23 year old lady is undergoing a trendelenberg procedure for varicose veins.
During the dissection of the saphenofemoral junction, which of the structures listed
below is most liable to injury?
A. Superficial circumflex iliac artery
B. Superficial circumflex iliac vein
C. Femoral artery
D. Femoral nerve
E. Deep external pudendal artery

Theme from September 2011 exam

85. Which of the following nerves is responsible for the motor innervation of the
sternocleidomastoid muscle?
A. Ansa cervicalis
B. Accessory nerve
C. Hypoglossal nerve
D. Facial nerve
E. Vagus nerve

Theme from January 2013 Exam

86. A 42 year old lady has had an axillary node clearance for breast malignancy. Post
operatively she reports weakness of the shoulder. She is unable to push herself forwards
from a wall with the right arm and the scapula is pushed out medially from the chest wall.
What is the most likely nerve injury?
A. C5, C6
B. C8, T1
C. Axillary nerve
D. Long thoracic nerve
E. Spinal accessory nerve

Theme from January 2012 and 2009 Exam

87. A 36 year old male is admitted for elective surgery for a lymph node biopsy in the
supraclavicular region. Post operatively the patient has difficulty shrugging his left
shoulder. What nerve has been damaged?
A. Phrenic nerve

19
B. Axillary nerve
C. C5, C6 lesion
D. C8, T1 lesion
E. Accessory nerve
Theme from September 2011 Exam

88. A 17 year old male presents to the clinic. He complains of difficulty using his left
hand. It has been a persistent problem since he sustained a distal humerus fracture as a
child. On examination there is diminished sensation overlying the hypothenar eminence
and medial one and half fingers. What is the most likely nerve lesion?
A. Anterior interosseous nerve
B. Posterior interosseous nerve
C. Ulnar nerve
D. Median nerve
E. Radial nerve
Theme from April 2012 Exam

89. A 56 year old man is undergoing a pancreatectomy for carcinoma. During resection
of the gland which of the following structures will the surgeon not encounter posterior to
the pancreas itself?
A. Left crus of the diaphragm
B. Superior mesenteric vein
C. Common bile duct
D. Portal vein
E. Gastroduodenal artery
Theme from 2010 Exam

90. Which of the following bones is related to the cuboid at its distal articular surface?
A. All metatarsals
B. 5th metatarsal
C. Calcaneum
D. Medial cuneiform
E. 3rd metatarsal
Theme from April 2012 Exam

91. A 34 year old lady presents with symptoms of faecal incontinence. Ten years
previously she gave
birth to a child by normal vaginal delivery. Injury to which of the following nerves is
most likely to account for this process?
A. Genitofemoral
B. Ilioinguinal
C. Pudendal
D. Hypogastric autonomic nerve
E. Obturator
Theme from April 2012 Exam

20
92. A 56 year old man undergoes an abdomino-perineal excision of the rectum. He is
assessed in the outpatient clinic post operatively. His wounds are well healed. However,
he complains of impotence. Which of the following best explains this problem?
A. Sciatic nerve injury
B. Damage to the internal iliac artery
C. Damage to the hypogastric nerve plexus
D. Damage to the vas
E. Damage to the genitofemoral nerve
Theme from 2012 Exam

93. A woman develops winging of the scapula following a Patey mastectomy. What is the
most likely cause?
A. Division of pectoralis minor to access level 3 axillary nodes
B. Damage to the brachial plexus during axillary dissection
C. Damage to the long thoracic nerve during axillary dissection
D. Division of the thoracodorsal trunk during axillary dissection
E. Damage to the thoracodorsal trunk during axillary dissection
Theme from January 2012 exam

94. Which nerve directly innervates the sinoatrial node?


A. Superior cardiac nerve
B. Right vagus nerve
C. Left vagus nerve
D. Inferior cardiac nerve
E. None of the above
Theme from September 2011,September 2012 Exam

95. A 30 year old man presents with back pain and the surgeon tests the ankle reflex.
Which of the following nerve roots are tested in this manoeuvre?
A. S3 and S4
B. L4 and L5
C. L3 and L4
D. S1 and S2
E. S4 only
Theme from April 2012 Exam

96. An 18 year old male presents to casualty with a depressed skull fracture. This is
managed surgically. Over the next few days he complains of double vision on walking
down stairs and reading. On examination the left eye cannot look downwards and
medially. Which of the nerves listed below is most likely to be responsible?
A. Facial
B. Oculomotor
C. Abducens
D. Trochlear
E. Trigeminal nerve

21
Theme from September 2012 Exam

97. A 16 year old boy is hit by a car and sustains a blow to the right side of his head. He
is initially conscious but on arrival in the emergency department is comatose. On
examination his right pupil is fixed and dilated. The neurosurgeons plan immediate
surgery. What type of initial approach should be made?
A. Left parieto-temporal craniotomy
B. Right parieto-temporal craniotomy
C. Posterior fossa craniotomy
D. Left parieto-temporal burr holes
E. None of the above
Theme from April 2012 Exam

98. A 56 year old man is having a long venous line inserted via the femoral vein into the
right atrium for CVP measurements. The catheter is advanced through the IVC. At which
of the following levels does this vessel enter the thorax?
A. L2
B. T10
C. L1
D. T8
E. T6

Theme from 2010 Exam

99. A 23 year old man falls and injures his hand. There are concerns that he may have a
scaphoid fracture as there is tenderness in his anatomical snuffbox on clinical
examination. Which of the following forms the posterior border of this structure?
A. Basilic vein
B. Radial artery
C. Extensor pollicis brevis
D. Abductor pollicis longus
E. Extensor pollicis longus
Theme from 2009,September 2012 Exam

100. The integrity of which muscle is assessed by the Trendelenberg test?


A. Sartorius
B. Quadratus femoris
C. Semimembranosus
D. Gluteus medius
E. Piriformis
Theme from 2011 Exam

22
101. Which of the following regions of the male urethra is entirely surrounded by Bucks
fascia?
A. Preprostatic part
B. Prostatic part
C. Membranous part
D. Spongiose part
E. None of the above
Theme from 2010 Exam

102. A 73 year old man has a large abdominal aortic aneurysm. During a laparotomy for
planned surgical repair the surgeons find the aneurysm is far more proximally located and
lies near the origin of the SMA. During the dissection a vessel lying transversely across
the aorta is injured. What is this vessel most likely to be?
A. Left renal vein
B. Right renal vein
C. Inferior mesenteric artery
D. Ileocolic artery
E. Middle colic artery
Theme from April 2012 Exam

103. A 56 year old machinist has his arm entrapped in a steel grinder and is brought to
the emergency department. On examination, he is unable to extend his
metacarpophalangeal joints and abduct his shoulder. He has weakness of his elbow and
wrist. What has been injured?
A. Ulnar nerve
B. Axillary nerve
C. Medial cord of brachial plexus
D. Lateral cord of brachial plexus
E. Posterior cord of brachial plexus
The posterior cord gives rise to:
Radial nerve ((innervates the triceps, brachioradialis, wrist extensors, and finger
extensors). Axillary nerve (innervates deltoid and teres minor). Upper subscapular nerve
(innervates subscapularis). Lower subscapular nerve (innervates teres major and
subscapularis). Thoracodorsal nerve (innervates latissimus dorsi).
(September 2012 exam)

104. A 60 year old female is undergoing a Whipples procedure for adenocarcinoma of the
pancreas. As the surgeons begin to mobilise the pancreatic head they identify a large
vessel passing inferiorly over the anterior aspect of the pancreatic head. What is it likely
to be?
A. Superior mesenteric artery
B. Coeliac axis
C. Inferior mesenteric artery
D. Aorta
E. Left gastric artery
Theme from January 2012 Exam

23
105. Which of the following structures does not lie posterior to the right kidney?
A. Psoas major
B. Transversus abdominis
C. Quadratus lumborum
D. Medial artcuate ligament
E. 10th rib
Theme from April 2012 Exam

106. Which of the following muscles is not within the posterior compartment of the lower
leg?
A. Peroneus brevis
B. Flexor digitalis longus
C. Soleus
D. Popliteus
E. Flexor hallucis longus
Theme from 2007 Exam

107. A 20 year old man undergoes an open appendicectomy performed via a lanz incision.
This surgeon places the incision on a level of the anterior superior iliac spine in an
attempt to improve cosmesis. During the procedure the appendix is found to be
retrocaecal and the incision is extended laterally. Which of the following nerves is at
greatest risk of injury?
A. Genitofemoral
B. Ilioinguinal
C. Obturator
D. Lateral femoral cutaneous
E. Femoral
Theme from April 2012 Exam

108. A 32 year old man is stabbed in the neck and the inferior trunk of his brachial plexus
is injured. Which of the modalities listed below is least likely to be affected?
A. Initiating abduction of the shoulder
B. Abduction of the fingers
C. Flexion of the little finger
D. Sensation on the palmar aspect of the little finger
E. Gripping a screwdriver
Inferior trunk of brachial plexus- C8 and T1 roots. Contributes to ulnar nerve and part of
median nerve. (Theme from September 2012 Exam)

109. A 23 year old man presents with delayed diagnosis of appendicitis. The appendix is
retrocaecal and has perforated causing a psoas abscess. Into which structure does the
psoas major muscle insert?
A. Greater trochanter of the femur
B. Linea aspera of the femur
C. Lesser trochanter of the femur
D. Iliac crest

24
E. None of the above
Theme based on 2011 exam

110. A 63 year old man is due to undergo a splenectomy. Which splenic structure lies
most posteriorly?
A. Gastrosplenic ligament
B. Splenic vein
C. Splenic artery
D. Splenic notch
E. Lienorenal ligament
Theme from 2011 Exam

111. A 62 year old man presents with arm weakness. On examination he has a weakness
of elbow extension and loss of sensation on the dorsal aspect of the first digit. What is the
site of the most likely underlying defect?
A. Axillary nerve
B. Median nerve
C. Ulnar nerve
D. Radial nerve
E. Musculocutaneous nerve
Theme from April 2012 Exam

112. In relation to the middle cranial fossa, which of the following statements relating to
the foramina are incorrect?
A. The foramen rotundum transmits the maxillary nerve
B. The foramen lacerum transmits the internal carotid artery
C. The foramen spinosum lies posterolateral to the foramen ovale
D. The foramen ovale transmits the middle meningeal artery
E. The foramen rotundum lies anteromedial to the foramen ovale
Theme addressed in 2010 and 2011 exam

113. During an operation for varicose veins the surgeons are mobilising the long
saphenous vein. Near its point of entry to the femoral vein an artery is injured and
bleeding is encountered. From where is the bleeding most likely to originate?
A. Femoral artery
B. Profunda femoris artery
C. Superficial circumflex iliac artery
D. Superficial epigastric artery
E. Deep external pudendal artery
Theme from 2011 Exam

114. What is embryological origin of the pulmonary artery?


A. First pharyngeal arch
B. Second pharyngeal arch
C. Fourth pharyngeal arch
D. Fifth pharyngeal arch

25
E. Sixth pharyngeal arch
Theme from September 2011,September 2012 Exam

115. A 53 year old lady presents with pain and discomfort in her hand. She works as a
typist and notices that the pain is worst when she is working. She also suffers symptoms
at night. Her little finger is less affected by the pain. Which of the nerves listed below is
most likely to be affected?
A. Radial
B. Median
C. Ulnar
D. Anterior interosseous nerve
E. Posterior interosseous nerve

Motor supply: LOAF


L ateral 2 lumbricals
O pponens pollicis
A bductor pollicisbrevis
F lexor pollicis brevis
Theme from April 2012 Exam

116. Which of the following muscles lies medial to the long thoracic nerve?
A. Serratus anterior
B. Latissimus dorsi
C. Pectoralis major
D. Pectoralis minor
E. None of the above
Theme from 2009 Exam

117. A 25 year old man is being catheterised, prior to a surgical procedure. As the
catheter enters the prostatic urethra which of the following changes will occur?
A. Resistance will increase significantly
B. Resistance will increase slightly
C. It will lie horizontally
D. Resistance will decrease
E. It will deviate laterally
Theme from September 2011 Exam

118. Which of the following nerves is the primary source of innervation to the anterior
scrotal skin?
A. Genital branch of the genitofemoral nerve
B. Pudendal nerve
C. Ilioinguinal nerve
D. Femoral branch of the genitofemoral nerve
E. Obturator nerve

Theme from April 2012 Exam

26
119. A 38 year old man presents to the clinic with shoulder weakness. On examination he
has an inability to initiate shoulder abduction. Which of the nerves listed below is least
likely to be functioning normally?
A. Suprascapular nerve
B. Medial pectoral nerve
C. Axillary nerve
D. Median nerve
E. Radial nerve
Theme from April 2012 Exam

120. A 72 year old lady with osteoporosis falls and sustains an intracapsular femoral neck
fracture. The fracture is completely displaced. Which of the following vessels is the main
contributor to the arterial supply of the femoral head?
A. Deep external pudendal artery
B. Superficial femoral artery
C. External iliac artery
D. Circumflex femoral arteries
E. Superficial external pudendal artery
Theme from 2010 Exam

121. A 21 year old man is hit with a hammer and sustains a depressed skull fracture at the
vertex. Which of the following sinuses is at risk in this injury?
A. Superior sagittal sinus
B. Inferior petrosal sinus
C. Transverse sinus
D. Inferior sagittal sinus
E. Straight sinus
Theme in September 2011 Exam

122. A 19 year old man is playing rugby when he suddenly notices a severe pain at the
posterolateral aspect of his right thigh. Which of the following muscle groups is most
likely to have been injured?
A. Semimembranosus
B. Semitendinosus
C. Long head of biceps femoris
D. Gastrocnemius
E. Soleus
Theme from April 2012 Exam

123. A 23 year old is stabbed in the groin and develops hypovolaemic shock. What is the
most likely finding on analysis of his urine?
A. Decreased specific gravity
B. Increased specific gravity
C. Increased urinary glucose
D. Increased urinary protein
E. Increased red blood cells in the urine (Theme from April 2013 Exam)

27
124. A 45 year old male is diagnosed with carcinoma of the head of the pancreas. He
reports that his stool sticks to the commode and will not flush away. Loss of which of the
following enzymes is most likely to be responsible for this problem?
A. Lipase
B. Amylase
C. Trypsin
D. Elastase
E. None of the above
Theme from April 2012 Exam

125. A 34 year old man receives morphine following an appendicectomy. He develops


constipation as a result. Which of the following best accounts for this process?
A. Stimulation of DOPA receptors
B. Inhibition of DOPA receptors
C. Stimulation of receptors
D. Stimulation of serotonin release
E. Inhibition of serotonin release
4 Types of opioid receptor:
(located in CNS)- Accounts for analgesic and antidepressant effects
k (mainly CNS)- analgesic and dissociative effects
(central and peripheral) - causes analgesia, miosis, decreased gut motility
Nociceptin receptor (CNS)- Affect of appetite and tolerance to agonists.
Theme from April 2013 Exam

126. A 56 year old male presents to the acute surgical take with severe abdominal pain.
He is normally fit and well. He has no malignancy. The biochemistry laboratory contacts
the ward urgently, his corrected calcium result is 3.6 mmol/l. What is the medication of
choice to treat this abnormality?
A. IV Pamidronate
B. Oral Alendronate
C. Dexamethasone
D. Calcitonin
E. IV Zoledronate
Theme from January 2012 exam

127. An over enthusiastic medical student decides to ask you questions about ECGs.
Rather than admitting your dwindling knowledge on this topic, you bravely attempt to
answer her questions! One question is what segment of the ECG represents ventricular
repolarization?
A. QRS complex
B. Q-T interval
C. P wave
D. T wave
E. S-T segment
Theme from January 2012 exam

28
128. A 28 year old man is shot in the abdomen and haemorrhages. Which of the
following substances will produce vasoconstriction in response to this process?
A. Renin
B. Angiotensin I
C. Angiotensin II
D. Aldosterone
E. None of the above
Similar theme to September 2011 Exam

129. A 43 year old lady is recovering on the intensive care unit following a Whipples
procedure. She has a central venous line in situ. Which of the following will lead to the
"y" descent on the waveform trace?
A. Ventricular contraction
B. Emptying of the right atrium
C. Emptying of the right ventricle
D. Opening of the pulmonary valve
E. Cardiac tamponade
JVP
3 Upward deflections and 2 downward deflections
Upward deflections
a wave = atrial contraction.
c wave = ventricular contraction
v wave = atrial venous filling
Downward deflections
x wave = atrium relaxes and tricuspid valve moves down
y wave = ventricular filling
Theme from January 2012

130. A 25 year old man is undergoing respiratory spirometry. He takes a maximal


inspiration and maximally exhales. Which of the following measurements will best
illustrate this process?
A. Functional residual capacity
B. Vital capacity
C. Inspiratory capacity
D. Maximum voluntary ventilation
E. Tidal volume
Theme from April 2012 Exam

131. What is the typical stroke volume in a resting 70 Kg man?


A. 10ml
B. 150ml
C. 125ml
D. 45ml
E. 70ml
Theme from April 2012 Exam

29
132. A 25 year old man undergoes an appendicetomy for appendicitis. The appendix is
submitted for histopathological evaluation. Which of the following is most likely to be
identified microscopically?
A. Macrophages
B. Neutrophils
C. Fibroblasts
D. Lymphocytes
E. Stem cells
Theme from April 2013 Exam

133. A 73 year old man has an arterial line in situ. On studying the trace the incisura can
be seen. What is the physiological event which accounts for this process?
A. Atrial repolarisation
B. Mitral valve closure
C. Ventricular repolarisation
D. Elastic recoil of the aorta
E. Tricuspid valve closure
Theme from 2010 Exam
It is the temporary rise in aortic pressure occurring as a result of elastic recoil

134. A 22 year old man suffers a blunt head injury. He is drowsy and has a GCS of 7 on
admission. Which of the following is the major determinant of cerebral blood flow in this
situation?
A. Systemic blood pressure
B. Mean arterial pressure
C. Intra cranial pressure
D. Hypoxaemia
E. Acidosis
Theme from 2009 Exam
Hypoxaemia and acidosis may both affect cerebral blood flow. However, in the traumatic
situation increases in intracranial pressure are far more likely to occur especially when
GCS is low. This will adversely affect cerebral blood flow.

135. A 43 year old man has recurrent episodes of dyspepsia and treatment is commenced
with oral antacids. Which of the hormones listed below is released in response to
increased serum gastrin levels and decreases intra gastric pH?
A. Cholecystokinin
B. Histamine
C. Somatostatin
D. Insulin
E. Vasoactive intestinal peptide
Theme from January 2013 Exam

30
136. Which of the following cell types is least likely to be found in a wound 1 week
following injury?
A. Macrophages
B. Fibroblasts
C. Myofibroblasts
D. Endothelial cells
E. Neutrophils
Theme from April 2012 Exam

137. Which of the following best accounts for the action of PTH in increasing serum
calcium levels?
A. Activation of vitamin D to increase absorption of calcium from the
small intestine.
B. Direct stimulation of osteoclasts to absorb bone with release of
calcium.
C. Stimulation of phosphate absorption at the distal convoluted tubule of
the kidney.
D. Decreased porosity of the vessels at Bowmans capsule to calcium.
E. Vasospasm of the afferent renal arteriole thereby reducing GFR and
calcium urinary loss.
Theme from April 2012 Exam
PTH increases the activity of 1--hydroxylase enzyme, which converts 25-
hydroxycholecalciferol to 1,25-dihydroxycholecalciferol, the active form of vitamin D.
Osteoclasts do not have a PTH receptor and effects are mediated via osteoblasts.

138. What is the most likely cause for this patients deterioration?
A. Acute respiratory alkalosis secondary to hyperventilation
B. Over administration of oxygen in a COPD patient
C. Metabolic acidosis secondary to severe pancreatitis
D. Metabolic alkalosis secondary to hypokalaemia
E. Acute respiratory acidosis secondary to pneumonia
Theme from April 2012 exam

139. A 43 year old man has a nasogastric tube inserted. The nurse takes a small aspirate
of the fluid from the stomach and tests the pH of the aspirate. What is the normal
intragastric pH?
A. 0.5
B. 2
C. 4
D. 5
E. 6
Theme from January 2012 Exam

31
140. A 22 year old lady receives intravenous morphine for acute abdominal pain. Which
of the following best accounts for its analgesic properties?
A. Binding to opioid receptors in the brainstem
B. Binding to opioid receptors at peripheral nerve sites
C. Binding to opioid receptors within the CNS
D. Binding to opioid receptors within the CNS
E. Binding to opioid receptors within the CNS
4 Types of opioid receptor:
(located in CNS)- Accounts for analgesic and antidepressant effects
k (mainly CNS)- analgesic and dissociative effects
(central and peripheral) - causes analgesia, miosis, decreased gut motility
Nociceptin receptor (CNS)- Affect of appetite and tolerance to agonists.
Theme from April 2012 Exam

141. Which of the following areas is predominantly concerned with thermoregulation?


A. Hypothalamus
B. Anterior pituitary
C. Cerebellum
D. Brain stem
E. Temporal lobe
Theme from 2012 Exam

142. A 19 year old man is attacked outside a club and beaten with a baseball bat. He
sustains a blow to the right side of his head. He is brought to the emergency department
and a policy of observation is adopted. His glasgow coma score deteriorates and he
becomes comatose. Which of the following haemodynamic parameters is most likely to
be present?
A. Hypertension and bradycardia
B. Hypotension and tachycardia
C. Hypotension and bradycardia
D. Hypertension and tachycardia
E. Normotension and bradycardia
Theme from April 2013 Exam

143. Which substance can be used to achieve the most accurate measurement of the
glomerular filtration rate?
A. Glucose
B. Protein
C. Inulin
D. Creatinine
E. Para-amino hippuric acid
Theme from January 2013 exam
Creatinine declines with age due to decline in renal function and muscle mass. Glucose,
protein (amino acids) and PAH are reabsorbed by the kidney.

32
144. A 17 year old lady with long standing anorexia nervosa is due to undergo excision
of a lipoma. Which of the following nutritional deficiencies is most likely to be
implicated in poor collagen formation as the wound heals?
A. Deficiency of copper
B. Deficiency of iron
C. Deficiency of ascorbic acid
D. Deficiency of phosphate
E. None of the above
Theme from January 2011

145. A 45 year old man is undergoing a small bowel resection. The anaesthetist decides
to administer an intravenous fluid which is electrolyte rich. Which of the following most
closely matches this requirement?
A. Dextrose / Saline
B. Pentastarch
C. Gelofusine
D. Hartmans
E. 5% Dextrose with added potassium 20mmol/ L
Theme from April 2012 Exam

146. A 16 year old girl develops pyelonephritis and is admitted in a state of septic shock.
Which of the following is not typically seen in this condition?
A. Increased cardiac output
B. Increased systemic vascular resistance
C. Oliguria may occur
D. Systemic cytokine release
E. Tachycardia

Cardiogenic Shock:e.g. MI, valve abnormality


1. Increased SVR (vasoconstriction in response to low BP)
2. Increased HR (sympathetic response)
3. Decreased cardiac output
4. Decreased blood pressure
Hypovolaemic shock:
1. Blood volume depletion
2. Haemorrhage, vomiting, diarrhea, dehydration, third-space losses during
major operations
3. Increased SVR
4. Increased HR
5. Decreased cardiac output
6. Decreased blood pressure
Septic shock:
1. Occurs when the peripheral vascular dilatation causes a fall in SVR
2. Similar response may occur in anaphylactic shock, neurogenic shock
3. Reduced SVR

33
4. Increased HR
5. Normal/increased cardiac output
6. Decreased blood pressure
Theme from January 2012 Exam
The SVR is reduced in sepsis and for this reason a vasoconstricting inotrope such as
noradrenaline may be used if hypotension and oliguria remain a concern despite
administration of adequate amounts of intravenous fluids.

147. Which receptor does noradrenaline mainly bind to?


A. 1 receptors
B. 2 receptors
C. 1 receptors
D. 2 receptors
E. G receptors
Theme from 2009 Exam

148. A 12 year old child is admitted with a 12 hour history of colicky right upper
quadrant pain. On examination the child is afebrile and is jaundiced. The abdomen is soft
and non tender at the time of examination. What is the most likely cause?
A. Infectious hepatitis
B. Acute cholecystitis
C. Cholangitis
D. Hereditary spherocytosis
E. Gilberts syndrome
Theme from September 2012 Exam

149. A 40 year old man presents with obstructive jaundice. Twenty years previously he
underwent a right hemicolectomy for a mucinous right sided colonic carcinoma. He was
subsequently diagnosed as having Lynch syndrome. What is the most likely cause of his
jaundice?
A. Hepatocellular carcinoma
B. Liver metastasis from colonic cancer
C. Pancreatic carcinoma
D. Duodenal carcinoma
E. Gastric carcinoma
Theme from April 2013

150. A 2 day old baby is noted to have voiding difficulties and on closer inspection is
noted to have hypospadias. Which of the following abnormalities is most commonly
associated with the condition?
A. Cryptorchidism
B. Diaphragmatic hernia
C. Ventricular - septal defect
D. Bronchogenic cyst
E. Atrial septal defect
Theme from January 2012 Exam

34
151. A 5 year old boy presents to the clinic with short stature suggestive of
achondroplasia. What is the genetic basis of this condition?
A. X linked defect
B. Y linked defect
C. YY linked defect
D. Autosomal dominant defect
E. Autosomal recessive defect
Theme from April 2013 Exam

152. A 58 year old man has been suffering from mechanical back pain for several years.
One morning he awakes from sleep and feels a sudden onset of pain in his back radiating
down his left leg. Which of the following events is most likely to account for his
symptoms?
A. Prolapse of inner annulus fibrosus
B. Prolapse of outer annulus fibrosus
C. Prolapse of nucleus pulposus
D. Rupture of the ligamentum flavum
E. None of the above
Theme from 2009 Exam, September 2012 Exam

153. A 34 year old man presents to the surgical clinic 8 months following a laparotomy
for a ruptured spleen. He complains of a nodule in the centre of his laparotomy wound.
This is explored surgically and a stitch granuloma is found and excised. From which of
the following cell types do granulomata arise?
A. Polymorpho nucleocytes
B. Plasma cells
C. Reed- Sternberg cells
D. Platelets
E. Macrophages
Granulomas are organised collections of macrophage. Theme from 2011 Exam

154. A 72 year old man has just undergone an emergency repair for a ruptured abdominal
aortic aneurysm. Pre operatively he was taking aspirin, clopidogrel and warfarin. Intra
operatively he received 5000 units of unfractionated heparin prior to application of the
aortic cross clamp. His blood results on admission to the critical care unit are as follows:
Full blood count Hb 8 g/dl, Platelets 40 * 109/l, WBC 7.1 * 109/l. His fibrin degradation
products are measured and found to be markedly elevated. Which of the following
accounts for these results?
A. Anastomotic leak
B. Disseminated intravascular coagulation
C. Heparin induced thrombocytopenia
D. Adverse effect of warfarin
E. Adverse effects of antiplatelet agents
Theme from April 2012 Exam

35
155. A 23 year old man presents to the surgical clinic with an inguinal hernia. On
examination he has a small direct hernia. However, you also notice that he has pigmented
spots around his mouth, on his palms and soles. In his history he underwent a reduction
of an intussusception aged 12 years. Which of the following lesions is most likely to be
identified if a colonoscopy were performed?
A. Hamartomas
B. Tubulovillous adenoma
C. Colorectal cancer
D. Crohns disease
E. Hyperplastic polyps
Theme from April 2012 Exam, January 2013 Exam
He is most likely to have Peutz-Jeghers syndrome which is associated with Hamartomas.

156. A 56 year old surgeon has been successfully operating for many years. Over the past
few weeks she has begun to notice that her hands are becoming blistering and weepy. A
latex allergy is diagnosed. Which of the following pathological processes accounts for
this scenario?
A. Type 1 hypersensitivity reaction
B. Type 2 hypersensitivity reaction
C. Type 4 hypersensitivity reaction
D. Type 3 hypersensitivity reaction
E. None of the above
Hypersensitivity reactions: ACID
type 1 --Anaphylactic
type 2 --Cytotoxic
type 3 --Immune complex
type 4 --Delayed hypersensitivity
Theme from 2012 Exam

157. A 43 year old man presents with haemoptysis and is diagnosed as having
tuberculosis. Which of the cell types listed below will usually internalise the tubercule
bacullis?
A. Fibroblast
B. Neutrophil
C. Erythrocyte
D. Macrophage
E. Eosinophil
Theme from January 2013 Exam

158. Which of these tumour markers is most helpful in identifying an individual with
hepatocellular carcinoma?
A. Serum AFP
B. Serum CA19-9
C. CEA
D. Beta HCG
E. CA125 Theme from September 2011 Exam

36
159. A pathologist is examining a histological section and identifies Hassall's corpuscles.
With what are they most commonly associated?
A. Follicular carcinoma of the thyroid
B. Medulla of the thymus
C. Medulla of the spleen
D. Medulla of the kidney
E. Fundus of the stomach
Theme from 2010 Exam, January 2013 Exam, April 2013 Exam

160. A 32 year old man is involved in a house fire and sustains extensive partial thickness
burns to his torso and thigh. Two weeks post operatively he develops oedema of both
lower legs. The most likely cause of this is:
A. Iliofemoral deep vein thrombosis
B. Venous obstruction due to scarring
C. Hypoalbuminaemia
D. Excessive administration of intravenous fluids
E. None of the above
Theme from 2009 Exam

161. A 45-year-old lady has recently undergone a thyroidectomy for treatment of


medullary thyroid cancer. Which of the following tumor markers is used clinically to
screen for recurrence?
A. Free T3
B. Thyroglobulin
C. Calcitonin
D. Free T4
E. Thyroid stimulating hormone
Theme from 2011 Exam

162. A 22 year old man is kicked in the head during a rugby match. He is temporarily
concussed, but then regains consciousness. Half an hour later he develops slurred speech,
ataxia and loses consciousnesss. On arrival in hospital he is intubated and ventilated. A
CT Scan is performed which shows an extradural hematoma. What is the most likely
cause?
A. Basilar artery laceration
B. Middle meningeal artery laceration
C. Laceration of the sigmoid sinus
D. Laceration of the anterior cerebral artery
E. Laceration of the middle cerebral artery
Theme based on September 2011, April 2013 Exam

37
163. A baby is born by normal vaginal delivery at 39 weeks gestation. Initially all
appears well and then the clinical staff become concerned because the baby develops
recurrent episodes of cyanosis. These are worse during feeding and improve dramatically
when the baby cries. The most likely underlying diagnosis is:
A. Choanal atresia
B. Oesophageal reflux
C. Tetralogy of Fallot
D. Oesophageal atresia
E. Congenital diaphragmatic hernia
Theme from 2011, April 2013 Exam

164. A 20 year old man is involved in a road traffic accident. Following the incident he is
unable to extend his wrist. However, this improves over the following weeks. Which type
of injury is he most likely to have sustained?
A. Radial nerve neurotmesis
B. Radial nerve neuropraxia
C. Axillary nerve axonotmesis
D. Ulnar nerve neuropraxia
E. Ulnar nerve axonotmesis
Theme from April 2011 Exam
Transient loss of function makes neuropraxia the most likely injury. The wrist extensors
are innervated by the radial nerve making this the most likely site of injury.

165. From which of the following cell types do giant cells most commonly originate?
A. Neutrophils
B. Myofibroblasts
C. Fibroblasts
D. Macrophages
E. Goblet cells
Theme from September 2011 and 2009 Exam

166. A 43 year old lady with hypertension is suspected of having a phaeochromocytoma.


Which of the following investigations is most likely to be beneficial in this situation?
A. Dexamethasone suppression test
B. Urinary 5-Hydroxyindoleacetic Acid (5-HIAA)
C. Histamine provocation test
D. Tyramine provocation test
E. Urinary vanillymandelic acid measurements
Theme from September 2011 Exam, September 2012 Exam

167. A 46 year old lady presents with symptoms of diarrhoea, weight loss of 10 Kg and a
skin rash of erythematous blisters involving the abdomen and buttocks. The blisters have
an irregular border and both intact and ruptured vesicles. What is the most likely
diagnosis?
A. Colonic adenocarcinoma
B. Pancreatic adenocarcinoma

38
C. Tropical sprue
D. Glucagonoma
E. Insulinoma
Theme from September 2011 Exam
Theme from September 2012 Exam
Glucagonoma is strongly associated with necrolytic migratory erythema.

168. A 56 year old man presents with symptoms of neuropathic facial pain and some
weakness of the muscles of facial expression on the right side. On examination he has a
hard mass approximately 6cm anterior to the right external auditory meatus. What is the
most likely diagnosis?
A. Pleomorphic adenoma
B. Adenocarcinoma
C. Mucoepidermoid carcinoma
D. Adenoid cystic carcinoma
E. Lymphoma
Theme from September 2011 Exam
The patient is most likely to have a malignant lesion within the parotid. Of the
malignancies listed; adenoid cystic carcinoma has the greatest tendency to perineural
invasion.

169. A 45 year old women with a thyroid carcinoma undergoes a total thyroidectomy.
The post operative histology report shows a final diagnosis of medullary type thyroid
cancer. Which of the tests below is most likely to be of clinical use in screening for
disease recurrence?
A. Serum CA 19-9 Levels
B. Serum thyroglobulin levels
C. Serum PTH levels
D. Serum calcitonin levels
E. Serum TSH levels
Theme from September 2012 Exam

170. A 56 year old man has undergone a radical nephrectomy. The pathologist bisects the
kidney and identifies a pink fleshy tumour in the renal pelvis. What is the most likely
disease?
A. Renal cell carcinoma
B. Transitional cell carcinoma
C. Angiomyolipoma
D. Phaeochromocytoma
E. Renal adenoma
Most renal tumours are yellow or brown in colour. TCC's are one of the few tumours to
appear pink.
Theme from April 2012

39
171. A 73 year old man presents with haemoptysis and is suspected of suffering from
lung cancer. On examination he has an enlarged supraclavicular lymph node. Which of
the following features is most likely to be present on histological examination?
A. Increased mitoses
B. Apoptosis
C. Barr Bodies
D. Multinucleate giant cells
E. Granuloma
Theme from 2011 Exam
Increased mitoses are commonly seen in association with malignant transformation of
cells. Apoptosis is not a common feature of metastatic cancer. Barr Bodies are formed
during X chromosome inactivation in female somatic cells.

172. Which of the following pathological explanations best describes the initial
pathological processes occurring in an abdominal aortic aneurysm in an otherwise well
65 year old, hypertensive male?
A. Loss of elastic fibres from the adventitia
B. Loss of collagen from the adventitia
C. Loss of collagen from the media
D. Loss of elastic fibres from the media
E. Decreased matrix metalloproteinases in the adventitia
Theme from April 2012 Exam, April 2013 Exam

173. A 28-year-old lady has a malignant melanoma removed from her calf. Which of the
following pathological criteria carries the greatest prognostic weighting?
A. Vascular invasion
B. Abnormal mitoses
C. Breslow thickness
D. Perineural invasion
E. Lymphocytic infiltrates
Theme from April 2012 Exam

174. A 20 year old girl presents with a thyroid cancer, she is otherwise well with no
significant family history. On examination she has a nodule in the left lobe of the thyroid
with a small discrete mass separate from the gland itself. Which of the following is the
most likely cause?
A. Follicular carcinoma
B. Anaplastic carcinoma
C. Medullary carcinoma
D. Papillary carcinoma
E. B Cell Lymphoma
Theme from September 2011 Exam

40
175. A 28 year old lady is breast feeding her first child. She presents with discomfort of
the right breast. Clinical examination demonstrates erythema and an area that is fluctuant.
Aspiration and culture of the fluid is most likely to demonstrate infection with which of
the following organisms?
A. Clostridium perfringens
B. Staphylococcus aureus
C. Streptococcus pyogenes
D. Staphylococcus epidermidis
E. Actinomycosis
Theme from 2013, 2010 and 2009 Exam
Staphylococcus aureus is the commonest cause. The infants mouth is usually the source
as it damages the nipple areolar

176. A 56 year old man is diagnosed as having a glioma. From which of the following
cell types do these tumours usually originate?
A. Astrocytes
B. Oligodendrocytes
C. Ependymal cells
D. Squamous cells
E. Neuroglial cells
Theme from January 2012 Exam
Gliomas originate from glial (otherwise known as neuroglial) cells. These serve a
structural function in the CNS. The tumours produced may resemble a number of CNS
cell types. Tumours are therefore named according to the cells they resemble rather than
the origin. Where this is not possible they are termed gliomas.

177. A 63 year old man finds that he has to stop walking after 100 yards due to bilateral
calf pain. He finds that bending forwards and walking up hill helps. He is able to ride a
bike without any pain. What is the most likely underlying cause?
A. Lumbar canal stenosis
B. Diabetic neuropathy
C. Aorto-iliac occlusion
D. Occlusion of the superficial femoral artery
E. Pelvic rheumatoid arthritis
Theme from April 2012 Exam, April 2013 Exam
The positional nature of the pain and the fact that improves with walking uphill makes an
underlying vascular aetiology far less likely.

178. A 73 year old lady is admitted for a laparoscopic cholecystectomy. During her pre-
operative assessment it is noted that she is receiving furosemide for the treatment of
hypertension. Approximately what proportion of the sodium that is filtered at the
glomerulus will be subsequently excreted?
A. Up to 25%
B. Upt to 75%
C. Between 3 and 5%
D. <2%

41
E. Between 1 and 2%
Theme from 2010 Exam

179. A 59 year old man presents with recurrent episodes of urinary sepsis. In his history
he mentions that he has suffered from recurrent attacks of left iliac fossa pain over the
past few months. He has also notices bubbles in his urine. He undergoes a CT scan which
shows a large inflammatory mass in the left iliac fossa. No other abnormality is detected.
The most likely diagnosis is:
A. Ulcerative colitis
B. Crohns disease
C. Mesenteric ischaemia
D. Diverticular disease
E. Rectal cancer
Theme from 2009 and 2011 Exam

180. A 63 year old male presents with several episodes of haematuria. He suffers from
COPD secondary to long term smoking. Which is the most likely underlying cause?
A. Renal cortical adenoma
B. Renal adenocarcinoma
C. Nephroblastoma
D. Transitional cell carcinoma of the bladder
E. Adenocarcinoma of the bladder
Theme from 2009 Exam
TCC is the most common subtype and is strongly linked to smoking. The important point
to note in this question is the term most likely as renal adenocarcinoma may produce
similar symptoms but is less likely.

181. A male infant is born prematurely at 34 weeks gestation by emergency cesarean


section. He initially appears to be stable. However, over the ensuing 24 hours he develops
worsening neurological function. Which of the following processes is most likely to have
occurred?
A. Extra dural haemorrhage
B. Sub dural haemorrhage
C. Sub arachnoid haemorrhage
D. Intraventricular haemorrhage
E. Arteriovenous malformation
Theme from April 2012 Exam

182. A 43 year old man presents with dyspepsia and undergoes an upper GI endoscopy.
During the procedure diffuse gastric and duodenal ulcers are identified. A Clo test
confirms the presence of Helicobacter pylori infection. What is the most likely
explanation for the ulcers?
A. Decreased gastric motility
B. Increased urease activity
C. Decreased release of mucous and bicarbonate
D. Decreased gastrin levels

42
E. Increased acid production
Theme from April 2011 Exam

H-Pylori has a number of pathological effects. In this question the main issue is by what
mechanism the organism is able to induce both gastric and duodenal ulceration. Without
modestly elevated acid levels, the duodenum would not undergo gastric metaplasia. H-
Pylori cannot colonise duodenal mucosa and therefore the development of ulcers at this
site can only occur in those who have undergone metaplastic transformation (mediated by
increased acidity).

183. A 25 year old male pedestrian is involved in a road traffic accident. He sustains
multiple injuries and is admitted to the intensive care unit, intubated and ventilated. Over
the next week he develops adult respiratory distress syndrome. What is the main reason
for hypoxaemia in this condition?
A. Increased lung compliance
B. Reduced diffusion
C. Reduced surfactant
D. Reduced elastase
E. Left to right shunt
Theme from 2011 Exam, January 2013 Exam
The diffuse lung injury, which is associated with loss of surfactant and increased elastase
release from neutrophils, results in fluid accumulation. This leads to reduced diffusion,
which is the main reason for hypoxaemia.

184. A 48 year old women presents with recurrent loin pain and fevers. Investigation
reveals a staghorn calculus of the left kidney. Infection with which of the following
organisms is most likely?
A. Staphylococcus saprophyticus
B. Proteus mirabilis
C. Klebsiella
D. E-Coli
E. Staphylococcus epidermidis
Theme from April 2012 Exam
Infection with Proteus mirabilis accounts for 90% of all proteus infections. It has a urease
producing enzyme. This will tend to favor urinary alkalinisation which is a relative per-
requisite for the formation of staghorn calculi.

185. In patients with an annular pancreas where is the most likely site of obstruction?
A. The first part of the duodenum
B. The second part of the duodenum
C. The fourth part of the duodenum
D. The third part of the duodenum
E. The duodeno-jejunal flexure
Theme from 2011 Exam
Theme from September 2012 Exam

43
186. A 53 year old man presents with dyspepsia. An upper GI endoscopy is performed
and Helicobacter pylori is identified. A duodenal ulcer is present in the first part of the
duodenum. Duodenal biopsies are taken and demonstrate epithelium that resembles cells
of the gastric antrum. Which of the following is the most likely explanation for this
process?
A. Hyperplasia of the crypts of Lieberkhun
B. Duodenal metaplasia
C. Duodenal dysplasia
D. Duodenal carcinoma
E. Hyptertrophy of Brunners glands
Metaplasia = cell type conversion Theme in January 2012 exam

187. A 73 year old man is recovering following an emergency Hartmans procedure


performed for an obstructing sigmoid cancer. The pathology report shows a moderately
differentiated adenocarcinoma that invades the muscularis propria, 3 of 15 lymph nodes
are involved with metastatic disease. What is the correct stage for this?
A. Astler Coller Stage B2
B. Dukes stage A
C. Dukes stage B
D. Dukes stage C
E. Dukes stage D Theme from September 2011
Exam

188. A 56 year old man is diagnosed with an abdominal aortic aneurysm and undergoes a
CT scan to asses the size of the aorta. During the course of his investigations a lesion of
the adrenal gland is identified. It measures 1.5 cm in diameter and the gland is otherwise
normal. What is the most likely diagnosis?
A. Adrenal gland metastasis
B. Adrenal gland arterio-venous malformation
C. Adrenal cyst
D. Phaeochromocytoma
E. Adrenal cortical adenoma
25% of all adrenal lesions >4cm in diameter are malignant (Theme from 2011 Exam)
Incidentalomas of the adrenal gland are common and represent the most likely lesion in
this scenario. Clearly the other lesions are all possibilities but are unlikely.

189. A 63 year old man has a history of claudication that has been present for many years.
He is recently evaluated in the clinic and a duplex scan shows that he has an 85% stenosis
of the superficial femoral artery. Two weeks later he presents with a 1 hour history of
severe pain in his leg. On examination he has absent pulses in the affected limb and it is
much cooler than the contra-lateral limb. Which process best accounts for this
presentation?
A. Thrombosis
B. Embolus
C. Atheroma growth
D. Sub intimal dissection

44
E. Anaemia Theme from April 2012 Exam
190. Which of the following processes facilitates phagocytosis?
A. Apoptosis
B. Opsonisation
C. Proteolysis
D. Angiogenesis
E. Necrosis
Theme from 2008 Exam

191. A 55 year old man undergoes a colonoscopy and a colonic polyp is identified. It has
a lobular appearance and is located on a stalk in the sigmoid colon. Which of the
processes below best accounts for this disease?
A. Apoptosis
B. Metaplasia
C. Dysplasia
D. Calcification
E. Degeneration
Theme from April 2012 Exam
Most colonic polyps described above are adenomas. These may have associated dysplasia.
The more high grade the dysplasia the greater the level of clinical concern.

192. A 70 year old male presents with painless frank haematuria. Clinical examination is
unremarkable. Routine blood tests reveal a haemoglobin of 18g/dl but are otherwise
normal. What is the most likely underlying diagnosis?
A. Squamous cell carcinoma of the bladder
B. Adenocarcinoma of the prostate
C. Adenocarcinoma of the kidney
D. Wilms tumour
E. Transitional cell carcinoma of the renal pelvis
Theme from April 2012 Exam
Polycythaemia is a recognised feature of renal cell carcinoma. Wilms tumours most
commonly occur in children.

193. A newborn infant is noted to have a unilateral cleft lip only. What is the most likely
explanation for this process?
A. Incomplete fusion of the second branchial arch
B. Incomplete fusion of the nasolabial muscle rings
C. Incomplete fusion of the first branchial arch
D. Incomplete fusion of the third branchial arch
E. Incomplete fusion of the secondary palate
Theme from April 2012 Exam
Unilateral isolated cleft lip represents a failure of nasolabial ring fusion. It is not related
to branchial arch fusion. Arch disorders have a far more profound phenotype and
malformation sequences.

45
194. Which of the following tumors are most likely to give rise to para-aortic nodal
metastasis early?
A. Ovarian
B. Bladder
C. Rectal
D. Caecal
E. Cervical
Theme from 2009 Exam
Ovarian tumours are supplied by the ovarian vessels, these branch directly from the aorta.
The cervix drains to the internal and external iliac nodes.

195. A 33-year-old man is involved in a road traffic accident. He is initially stable and
transferred to the accident and emergency department. On arrival he is catheterized. One
minute later he becomes hypotensive, with evidence of angioedema surrounding his penis.
What is the most likely explanation for this event?
A. Type V latex hypersensitivity reaction
B. Type IV latex hypersensitivity reaction
C. Type III latex hypersensitivity reaction
D. Type I latex hypersensitivity reaction
E. Type II latex hypersensitivity reaction
Theme from January 2013 exam, April 2012 Exam

196. A 52-year-old man with dyspepsia is found to have a duodenal ulcer. A CLO test is
taken and is positive. Which statement relating to the likely causative organism is false?
A. It is a gram negative organism
B. It lives only on gastric type mucosa
C. It may occupy areas of ectopic gastric metaplasia
D. In patients who are colonised there is commonly evidence of fundal gastritis on
Endoscopy
E. It produces a powerful urease that forms the basis of the CLO test
Helicobacter pylori accounts for >75% cases of duodenal ulceration. It may be diagnosed
with either serology, microbiology, histology or CLO testing. Helicobacter pylori rarely
produces any typical features on endoscopy. Where infection is suspected the easiest
course of action is to take an antral biopsy for CLO testing in the endoscopy suite.
Theme from January 2011 Exam

197. A 13-month old boy is brought to the surgical clinic by his mother because his left
testicle is not located in the scrotum. At which of the following sites would the testicle be
located if it were an ectopic testis?
A. Canalicular
B. Inguinal
C. External inguinal ring
D. Superficial inguinal pouch
E. High scrotal
Theme from September 2011 Exam, January 2012 Exam

46
Ectopic testes are those that come to lie outside the normal range of embryological
descent (i.e. in the superficial inguinal pouch). Other sites of ectopic testes include; base
of penis, femoral and perineal.

198. Splenectomy increases the risk of infection from all the following organisms except?
A. Pneumococcus
B. Klebsiella
C. Haemophilus influenzae
D. Staphylococcus aureus
E. Neisseria meningitides Theme from 2010 Exam

199. An 18 month old boy presents with recurrent urinary tract infections. As part of the
diagnostic work-up he is noted to have abnormal renal function. An ultrasound scan is
performed and shows bilateral hydronephrosis. What is the most likely underlying
diagnosis?
A. Urethral valves
B. Meatal stenosis
C. Hydronephrosis
D. Pelvico-ureteric junction obstruction
E. Benign prostatic hyperplasia Theme from April 2012 Exam
A posterior urethral valve is an obstructive, developmental uropathy that usually affects
male infants incidence 1 in 8000). Diagnostic features include bladder wall hypertrophy,
hydronephrosis and bladder diverticula.

200. A 78-year-old lady presents with a tender swelling in her right groin. On
examination there is a tender swelling that lies lateral to the pubic tubercle. It has a cough
impulse. What is the most likely underlying diagnosis?
A. Thrombophlebitis of the great saphenous vein
B. Femoral hernia
C. Thrombophlebitis of saphena varix
D. Inguinal hernia
E. Obturator hernia Theme from April 2012 Exam

201. A 22-year-old man undergoes a splenectomy for an iatrogenic splenic injury. On the
second post-operative day a full blood count is performed. Which of the following
components of the full blood count is the first to be affected?
A. Erythrocyte count
B. Reticulocyte count
C. Eosinophil count
D. Monocyte count
E. Lymphocyte count Theme from January 2012 Exam
The granulocyte and platelet count are the first to be affected following splenectomy.
Then reticulocytes increase. Although a lymphocytosis and monocytosis are reported,
these take several weeks to develop.

47
202. A 28 year old man is in the surgical intensive care unit. He has suffered a flail
chest injury several hours earlier and he was intubated and ventilated. Over the past
few minutes he has become increasingly hypoxic and is now needing increased
ventilation pressures. What is the most common cause?
A. Pulmonary embolism
B. Cardiac tamponade
C. Fat embolism
D. Tension pneumothorax
E. Adult respiratory distress syndrome
Theme from April 2011 Exam
A flail chest segment may lacerate the underlying lung and create a flap valve. A
tension pneumothorax can be created by intubation and ventilation in this situation.
Sudden hypoxia and increased ventilation pressure are clues.

203. A 56 year old lady presents with a pathological fracture of the proximal femur.
Which of the following primary sites is the most likely source of her disease?
A. Thyroid
B. Breast
C. Kidney
D. Endometrium
E. None of the above
Theme from September 2011 exam
The correct answer is breast, because the question asks for the most likely primary
site. Breast cancer is the commonest cause of lytic bone metastasis in women of this
age, especially from amongst those options given.

204. A 63 year old man undergoes a laparotomy and small bowel resection. Twelve
hours post operatively he is noted to have a decreased urine output. Which of the
hormones listed below is most likely to be responsible?
A. Cortisol
B. Atrial natriuretic hormone
C. Vasopressin
D. Insulin
E. Glucagon
Theme from January 2013 Exam
Vasopressin is released in increased quantities following most operative procedures
and will tend to cause water retention. Excessive administration of intravenous
fluids in an attempt to force a diuresis may cause fluid overload in post operative
patients for this reason.

205. A 72 year old man is due to undergo an oesophagectomy for malignancy. His
BMI is 17.5. What is the best feeding regime immediately following surgery?
A. Total parenteral nutrition.
B. Feeding jejunostomy
C. Feeding duodenostomy.
D. Liquid diet orally.
48
E. Soft solids orally.
Theme from April 2012 Exam
This patient has a condition causing poor absorption, loss of nutrients and high
metabolism. Enteral feeds should be used where possible and many surgeons will
site a jejunostomy for this purpose. Oral diet is not permitted following a resection
until the anastomosis has had time to heal.

206. A patient with tachycardia and hypotension is to receive inotropes. Which of


the following conditions are most likely to be treated with inotropes?
A. Hypovolaemic shock
B. Septic shock
C. Neurogenic shock
D. Cardiogenic shock
E. None of the above
Theme from April 2012 Exam
The term septic shock has a precise meaning and refers to refractory systemic
arterial hypotension in spite of fluid resuscitation. Patients will therefore usually
require inotropes. Individuals suffering from neurogenic shock will usually receive
intravenous fluids to achieve a mean arterial pressure of 90mmHg. If this target
cannot be achieved then these patients will receive inotropes. Hypovolaemic shock
requires fluids and the management of cardiogenic shock is multifactorial and
includes inotropes, vasodilators and intra-aortic balloon pumps

207. A 27 year old man is involved in a road traffic accident. He is seen in the
emergency department with chest pain. Clinical examination is essentially
unremarkable and he is discharged. He subsequently is found dead at home. What is
the most likely underlying injury?
A. Tracheobronchial tree injury
B. Traumatic aortic disruption
C. Cardiac laceration
D. Diaphragmatic rupture
E. Rupture of the oesophagus
Theme from January 2013 Exam
Aortic injuries that do not die at the scene may have a contained haematoma.
Clinical signs are subtle and the diagnosis may not be apparent on clinical
examination. Without prompt treatment the haematoma usually bursts and the
patient dies.

208. What is the opposite of relative risk?


A. P value
B. Odds ratio
C. Likelihood ratio
D. Absolute risk reduction
E. Number needed to treat
Theme from September 2012 Exam

49
209. A 53 year old man presents with an ulcerated mass at the anal verge. A biopsy
is taken and the histology demonstrates as squamous cell carcinoma. Infection with
which of the viruses below is most likely to have contributed to the development of
the condition?
A. Human papillomavirus 7
B. Human immunodeficiency virus 1
C. Human immunodeficiency virus 2
D. Human papillomavirus 16
E. Human T-lymphotropic virus 1
Theme from January 2013 exam
Infection with human papilloma virus 16 is a risk factor for the development of intra
epithelial dysplasia of the anal skin with subsequent increased risk of invasive
malignancy.

210. A 32 year old man is involved in a motorcycle accident and sustains a closed
unstable spiral tibial fracture. This is managed with an intramedullary nail. On
return to the ward he is noted to have increasing pain in the limb and on
examination the limb is swollen and tender with pain on passive stretching of the
toes. The most likely diagnosis is:
A. Tibial nerve neuropraxia
B. Displaced tibial nail
C. Compartment syndrome
D. Deep vein thrombosis
E. Sciatic nerve injury
Theme from 2009 Exam
Theme from September 2012 Exam
Severe pain in a limb should raise suspicions of compartment syndrome especially
in tibial fractures following fixation with intra medullary devices.

211. A 70-year-old female is admitted with a history of passing brown coloured


urine and abdominal distension. On examination she is clinically got large bowel
obstruction with central abdominal tenderness. She is maximally tender in the left
iliac fossa. The is no evidence of haemodynamic instability. What is the most
appropriate investigation?
A. Cystogram
B. Abdominal X-ray of the kidney, ureters and bladder
C. Computerised tomogram of the abdomen and pelvis
D. Flexible sigmoidoscopy
E. Barium enema
Theme from April 2013 Exam
This lady is most likely to have a colovesical fistula complicating diverticular disease
of the sigmoid colon. In addition, she may also have developed a diverticular
stricture resulting in large bowel obstruction. A locally advanced tumour of the
sigmoid colon may produce a similar clinical picture. The best investigation of this
acute surgical patient is an abdominal CT scan, this will demonstrate the site of the
disease and also supply regional information such as organ involvement and other
50
local complications such as a pericolic abscess. A barium enema would require
formal bowel preparation and this is contra indicated where large bowel
obstruction is suspected. A flexible sigmoidoscopy is unlikely to be helpful and the
air insufflated at the time of endoscopy may make the colonic distension worse. A
cystogram would provide only very limited information.

212. A 53 year old man presents with a full thickness external rectal prolapse.
Which of the following procedures would be the most suitable surgical option?
A. Rectopexy
B. Delormes
C. Altmeirs
D. Thirsch tape
E. Abdomino-perineal excision of the rectum
Theme from 2012 Exam
As this man is young and has full thickness prolapse a rectopexy is the most
appropriate procedure. It will give the lowest recurrence rates.

213. A 28 year old man is involved in a road traffic accident and sustains a flail chest
injury. On arrival in the emergency department he is hypotensive. On examination
he has an elevated jugular venous pulse and auscultation of the heart reveals quiet
heard sounds. What is the most likely diagnosis?
A. Pneumothorax
B. Myocardial contusion
C. Cardiac tamponade
D. Haemothorax
E. Ventricular septal defect
Theme from 2010 exam
The presence of a cardiac tamponade is suggested by Becks Triad:
1. Hypotension
2. Muffled heart sounds
3. Raised JVP

214. A 23 year old man is recovering from an appendicectomy. The operation was
complicated by the presence of perforation. He is now recovering on the ward.
However, his urine output is falling and he has been vomiting. Which of the
following intravenous fluids should be initially administered, pending analysis of his
urea and electrolyte levels?
A. Hartmans solution
B. Dextran 70
C. Pentastarch
D. Gelofusin
E. 5% Dextrose
Theme from January 2011 Exam
He will have sequestration of electrolyte rich fluids in the abdomen and gut lumen.
These are best replaced by use of Hartmans solution in the first instance.

51
215. A 32 year old man presents to the acute surgical unit with acute pancreatitis.
He suddenly becomes dyspnoeic and his saturations are 89% on air. A CXR shows
bilateral pulmonary infiltrates. His CVP pressure is 16mmHg. What is the most likely
diagnosis?
A. Pulmonary oedema
B. Pneumococcal pneumonia
C. Staphylococcal pneumonia
D. Pneumocystis carinii
E. Adult respiratory distress syndrome
Theme from January 2012 Exam
Acute pancreatitis is known to precipitate ARDS. ARDS is characterised by bilateral
pulmonary infiltrates and hypoxaemia. Note that pulmonary oedema is excluded by
the CVP reading < 18mmHg.

216. A 29 year old female undergoes a sub total thyroidectomy. Five days post
operatively the wound becomes erythematous and discharges pus. What is the most
likely causative organism?
A. Streptococcus pyogenes
B. Haemophilus influenzae
C. Pseudomonas aeruginosa
D. Staphylococcus aureus
E. Proteus mirabilis
Theme from 2011 Exam
In this setting Staphylococcus aureus Infection is the most likely cause. In the UK
between 2010 and 2011 the commonest cause of wound infection was enterobacter
infections (usually following cardiac or colonic surgery). 23% of infections were due
to Staph aureus, which fits the scenario above. Infection with the other organisms
including strep pyogenes was much rarer.

217. Which organism is most likely to cause osteomyelitis in children with sickle cell
disease?
A. Group A Streptococcus species
B. Staphylococcus aureus
C. Enterobacter
D. Salmonella species
E. Group B Streptococcus species
Theme from January 2013 Exam
Worldwide the Salmonella species remains the most common infecting organism in
children with osteomyelitis.

218. A 22 year old female is admitted to A&E with a splenic rupture. She has not
been involved in any trauma. What infection can cause spontaneous splenic
rupture?
A. Measles
B. Mumps
C. Influenza
52
D. Epstein-Barr virus
E. Rubella
Theme from 2009 and 2011 Exams
Epstein - Barr Virus may cause generalised lymphadenopathy. This may be
associated with splenomegaly. This enlargement has been associated with
spontaneous rupture.

219. A 23 year old man sustains a severe facial fracture and reconstruction is
planned. Which of the following investigations will facilitate pre-operative planning?
A. Mandibular tomography
B. Magnetic resonance scan of face
C. Skull X-ray
D. Computerised tomography of the head
E. Orthopantomogram
Theme from 2011 Exam
Significant facial fractures may have intracranial communication. CT scanning will
allow delineation of injury extent and 3D reconstruction images can be created. An
Orthopantomogram (OPT) will provide good images of mandible and surrounding
bony structures but will not give intracranial detail. A skull xray lacks the detail for
modern practice.

220. A 22 year old man presents with a 5 day history of sore throat, malaise and
fatigue. On examination he has a large peritonsillar abscess. What is the most likely
underlying infective organism?
A. Epstein Barr Virus
B. Streptococcus pyogenes
C. Cytomegalovirus
D. Moraxella catarrhalis
E. Streptococcus viridans
Similar theme in September 2011 Exam
Quinsy usually occurs as a result of bacterial tonsillitis and the most common cause
of bacterial tonsillitis is streptococcal organisms.

221. A 21 year old man falls down a ravine whilst skiing and is trapped for several
hours. He is finally brought to the emergency department profoundly hypothermic
with a core temperature of 29C. Which method is most effective at raising the core
temperature?
A. Re-warming with electric blankets
B. Increasing the room temperature
C. Instillation of warm intravesical fluid
D. Instillation of warmed rectal fluid
E. Instillation of warmed intra peritoneal fluid
Theme from 2011 Examination
Visceral cavity re-warming be it lung or abdomen (or both) provides rapid
rewarming. Only extracorporeal circulatory devices provide faster rates of re-
warming.
53
222. A 21 year old man is undergoing an inguinal hernia repair and receives a dose
of intravenous coamoxiclav. He is reported to have a penicillin allergy. Over the next
few minutes his vital signs are: Pulse - 130bpm, blood pressure- 60/40mmHg. What
is the first line treatment?
A. Hydrocortisone 100mg IV
B. Adrenaline 1:1000 IV
C. Chlorpheniramine 10mg IV
D. Adrenaline 1:1000 IM
E. Adrenaline 1:10000 IV
Theme from 2009 Exam
The first line treatment of anaphylactic shock is intra muscular adrenaline.

223. A 43 year old man has symptoms of carcinoid syndrome. Which of the
following is the most effective therapeutic agent in controlling the symptoms?
A. Atenolol
B. Octreotide
C. Glucagon
D. Somatostatin
E. Spironolactone
Theme from April 2012 Exam
Octreotide is the usual treatment for carcinoid syndrome. Somatostatin inhibits the
release of a number of gut hormones. Octreotide is the synthetic alternative to
somatostatin and thus the most appropriate therapeutic agent.

224. A 22 year old man is brought to the emergency department. He was found lying
unconscious on his right arm and it is evident that he has taken a temazepam
overdose. His right arm is mottled in colour and swollen, his hand is insensate and
stiff. What substance is most likely to be present in the urine in increased
quantities?
A. Protein
B. Haemoglobin
C. Myoglobin
D. Erythrocytes
E. Lymphocytes
Theme from 2011 Exam
This man is likely to have muscle death secondary to compartment syndrome. This
will result in muscle breakdown and release of myoglobin. This may accumulate in
the kidney and result in renal failure.

225. A 72 year old man is due to undergo an oesophagectomy for malignancy. His
BMI is 17.5. What is the best feeding regime immediately following surgery?
A. Total parenteral nutrition.
B. Feeding jejunostomy
C. Feeding duodenostomy.
D. Liquid diet orally.
E. Soft solids orally. Theme from April 2012 Exam
54
226. A 48 year old man is recovering on the high dependency unit following a long
and complex laparotomy. His preoperative medication includes an ACE inhibitor for
blood pressure control. For the past two hours he has been oliguric with a urine
output of 10ml/hr-1. What the most appropriate course of action?
A. Stop the ACE inhibitor
B. Administer a fluid challenge
C. Start an infusion of nor adrenaline
D. Administer intravenous frusemide
E. Insert a Swann-Ganz Catheter

Theme from April 2012 Exam


Hypovolaemia is the most likely cause for oliguria and a fluid challenge is the most
appropriate action. Blind administration of inotropes to hypovolaemic patients is
unwise, with the possible exception of cardiac patients.

227. A 8 year old boy with recurrent attacks of otitis media is suspected of
developing a glue ear. If his sound conduction is tested, which of the following is
most consistent with a unilateral middle ear effusion?
A. Negative Rinne's test on the ipsilateral side
B. Positive Rinne's test on the ipsilateral side
C. Positive Webers and Rinnes tests on the ipsilateral side
D. Positive Rinne's test on the contralateral side
E. Negative Webers test only on the contralateral side
Theme from 2008 Exam
Rinne's test will localise to the affected side (i.e. it is negative in conductive
deafness). In a positive Rinne's test sound heard by air conduction is better than
that conveyed by bone conduction. Reduction of both air and bone conduction in
equal measure is a feature of sensorineural hearing loss.

228. A 24 year old man is brought to the emergency department have suffered a
crush injury to his forearm. Assessment demonstrates that the arm is tender, red
and swollen. There is clinical evidence of an ulnar fracture and the patient cannot
move their fingers. Which is the most appropriate course of action?
A. Application of an external fixation device
B. Closed reduction
C. Debridement
D. Discharge and review in fracture clinic
E. Fasciotomy
Theme from April 2012
The combination of a crush injury, limb swelling and inability to move digits should
raise suspicion of a compartment syndrome that will require a fasciotomy

55
229. A 10 year old boy presents with symptoms of right knee pain. The pain has
been present on most occasions for the past three months and the pain typically
lasts for several hours at a time. On examination he walks with an antalgic gait and
has apparent right leg shortening. What is the most likely diagnosis?
A. Perthes Disease
B. Osteosarcoma of the femur
C. Osteoarthritis of the hip
D. Transient synovitis of the hip
E. Torn medial meniscus

Theme from September 2012 Exam


There are many causes of the irritable hip in the 10-14 year age group. Many of
these may cause both hip pain or knee pain. Transient synovitis of the hip the
commonest disorder but does not typically last for 3 months. An osteosarcoma
would not usually present with apparent limb shortening unless pathological
fracture had occurred. A slipped upper femoral epiphysis can cause a similar
presentation although it typically presents later and with different patient
characteristics.

230. An 8 year old child falls onto an outstretched hand and is brought to the
emergency department. He is examined by a doctor and a bony injury is cleared
clinically. He re-presents a week later with pain in his hand. What is the most likely
underlying injury?
A. Fracture of the distal radius
B. Fracture of the scaphoid
C. Dislocation of the lunate
D. Rupture of flexor pollicis longus tendon
E. Bennett's fracture
Theme from January 2013 Exam
Scaphoid fractures in children will usually involve the distal pole and are easily
missed. The initial clinical examination (and sometimes x-rays) may be normal and
repeated clinical examination and imaging is advised for this reason. Whilst the
other injuries may be sustained from a fall onto an outstretched hand they are less
likely to be overlooked on clinical examination. In the case of a Bennetts fracture,
the injury mechanism is less compatible with this type of injury.

231. A 19 year old soldier has just returned from a prolonged marching exercise and
presents with a sudden onset, severe pain, in the forefoot. Clinical examination
reveals tenderness along the second metatarsal. Plain x-rays are taken of the area,
these demonstrate callus surrounding the shaft of the second metatarsal. What is
the most likely diagnosis?
A. Stress fracture
B. Mortons neuroma
C. Osteochondroma
D. Acute osteomyelitis
E. Freiberg's disease Theme from 2011 Exam
56
A short history of pain together with clinical examination and radiological signs
affecting the second metatarsal favour a stress fracture. The fact that callus is
present suggests that immobilisation is unlikely to be beneficial. Freibergs disease is
an anterior metatarsalgia affecting the head of the second metarsal, it typically
occurs in the pubertal growth spurt. The initial injury was thought to be due to
stress microfractures at the growth plate. The key feature in the history which
distinguishes the injury as being stress fracture is the radiology. In Freibergs disease
the x-ray changes include; joint space widening, formation of bony spurs, sclerosis
and flattening of the metatarsal head.

232. A 2-day-old baby girl is noted to become cyanotic whilst feeding and crying. A
diagnosis of congenital heart disease is suspected. What is the most likely cause?
A. Transposition of the great arteries
B. Coarctation of the aorta
C. Patent ductus arteriosus
D. Tetralogy of Fallot
E. Ventricular septal defect
Congenital heart disease- Cyanotic: TGA most common at birth, Fallot's
most common overall and Acyanotic: VSD most common cause
Theme from April 2011 exam

233. A 6 month old boy is brought to the clinic by his mother. She is concerned that
his testes are not located into the scrotum. She has noticed them only when he is in
the bath, but not at any other time. What is the most likely underlying diagnosis?
A. Rectractile testis
B. Ectopic testis
C. Undescended testis
D. Testicular agenesis
E. Intersex child Theme from April 2012 Exam
A testis that appears in warm conditions or which can be brought down on clinical
examination and does not immediately retract is usually a retractile testis.

234. A 22 year old man presents with an infected sebaceous cyst. The cyst itself is
swollen, discharging pus and has some surrounding erythema. What is the most
appropriate treatment?
A. Excision of the cyst of closure of the defect with interrupted 3/0 silk
B. Excision of the cyst and closure of the defect with subcuticular 4/0 undyed
nylon
C. Incision and drainage with excision of the cyst wall and packing of the
defect
D. Incision and drainage with conservation of the cyst wall and packing of the
defect
E. Administration of oral co-amoxyclav and definitive surgery once the
infection has cleared
January 2013 Exam
57
The correct treatment for an infected sebaceous cyst is incision and drainage with
removal of the cyst wall. Conservation of the cyst wall will invariably lead to
recurrence. Under no circumstances should an infected wound like this be primarily
closed. The administration of antibiotics without drainage of sepsis is futile.

235. A 21 year old man is involved in a road traffic accident. After a transient period
of concussion he is found to have a GCS of 15 by the paramedics. On arrival at
hospital he is monitored in a side room of the emergency department. When he is
next observed he is noted to have a GCS of 3 and a blown right pupil. Which of the
processes below best accounts for this deterioration?
A. Hydrocephalus
B. Intraventricular bleed
C. Sub dural bleed
D. Trans tentorial herniation
E. Sub arachnoid haemorrhage Theme from April 2012 Exam

The presence of a blown right pupil is a sign of a third cranial nerve compression.
The most likely cause is an extradural bleed. However, since this option is not listed
the process of trans tentorial herniation would be the most applicable answer.
Intraventricular bleeds are typically more common in premature neonates,
deterioration due to hydrocephalus is more chronic.

236. A 33 year old lady develops a thunderclap headache and collapses. A CT scan
shows that she has developed a subarachnoid haemorrhage. She currently has no
evidence of raised intracranial pressure. Which of the following drugs should be
administered?
A. None
B. Atenotol
C. Labetolol
D. Nimodipine
E. Mannitol
Theme from 2007 Exam
Nimodipine is a calcium channel blocker. It reduces cerebral vasospasm and
improves outcomes. It is administered to most cases of sub arachnoid haemorrhage.

237. A 28 year old women collapses and is found to be deeply comatose with a GCS
of 3. She is admitted to hospital, intubated and ventilated. A CT scan shows an
extensive sub arachnoid haemorrhage. A urinary catheter is inserted. What type of
urine output would be expected?
A. 30ml concentrated urine per hour
B. 30ml diluted urine per hour
C. 60ml diluted urine per hour
D. 300ml concentrated urine per hour
E. 300ml diluted urine per hour
Theme from 2011 Exam

58
An extensive CNS insult is likely to result in centrally mediated diabetes insipidus.
This may either be the result of hypothalamic injury directly or the result of
disruption of the hypothalamic-pituitary connections. The result is a large volume
diuresis with diluted urine.

238. A 49 year old female is due to undergo a renal transplant. Apart from ABO
compatibility which of the
following is most important in matching donor and recipient organs?
A. HLA-DR
B. Rhesus
C. HLA- A
D. HLA-B
E. Kidd Theme from January 2013 Exam
The effect of HLA-DR mismatches are the most clinically significant, since HLA-DR
mismatch increases graft loss five fold. HLA-B increases graft loss three fold and
HLA-A increases the risk two fold. Rhesus is not used to match organs to recipients.
Kidd is a minor group and of no significance.

239. A 45 year old man undergoes an upper gastrointestinal endoscopy for a benign
oesophageal stricture. This is dilated and he suffers an iatrogenic perforation at the
site. His imaging shows a small contained leak and a small amount of surgical
emphysema. What is the most appropriate nutritional option?
A. Nil by mouth and intravenous fluids alone
B. Intravenous fluids and sips orally
C. Total parenteral nutrition
D. Nasogastric feeding
E. PEG tube feeding Theme from April 2013 Exam
Iatrogenic perforations of the oesophagus may be managed non operatively. This
usually involves a nil by mouth regime, tube thoracostomy may be needed. Total
parenteral nutrition is the safest option. Insertion of NG feeding tubes and PEG
tubes may complicate the process or allow feed to enter the perforation site.

240. A 58 year old lady has a two year history of worsening dysphagia. In addition to
this she has a nocturnal cough. Over this time she has lost a total of 8kg in weight.
Which of the processes below is the most likely explanation for this?
A. Gastro-oesophageal reflux disease
B. Barretts oesophagus
C. Carcinoma
D. Mallory Weiss tear
E. Achalasia Theme from April 2013 Exam
Such marked weight loss should arouse suspicion of cancer. She is most likely to
have a longstanding stricture associated with oesophagitis and Barretts oesophagus.
This may progress to carcinoma which will tend to occur in association with
worsening dysphagia and weight loss.

59
241. A 22 year old man is involved in a road traffic accident. He is found to have a
pelvic fracture. While on the ward the nursing staff report that he is complaining of
lower abdominal pain. On examination you find a distended tender bladder. What is
the diagnosis?
A. Bladder rupture
B. Ureter injury
C. Urethral injury
D. Clot retention
E. Prostate rupture
Theme from 2009 Exam
Pelvic fractures may cause laceration of the urethra. Urinary retention, blood at the
urethral meatus and a high riding prostate on digital rectal examination are the
typical features.

242. A 33 year old lady attends the clinic with a 3 month history of palpitations and
irritability. Her thyroid function, PTH and calcium are measured: Thyroid function
Free T4 40 pmol/L, TSH < 0.1 miu/L, Free T3 25 p mol/L (normal 3.5-7.7 p mol/L)
PTH 10pg/ml (normal 10-55pg/ml) (Normal values listed in reference range link)
What is the most likely diagnosis?
A. Hypothyroidism
B. Hyperthyroidism
C. Hypoparathyroidism
D. Hyperparathyroidism
E. Euthyroid
Theme from April 2012 Exam
Elevated T4 and suppressed TSH makes this the most likely diagnosis. The PTH level
is normal.

60

Potrebbero piacerti anche